LSAT and Law School Admissions Forum

Get expert LSAT preparation and law school admissions advice from PowerScore Test Preparation.

 Administrator
PowerScore Staff
  • PowerScore Staff
  • Posts: 8916
  • Joined: Feb 02, 2011
|
#72932
Complete Question Explanation

Justify the Conclusion. The correct answer choice is (D).

Answer choice (A):

Answer choice (B):

Answer choice (C):

Answer choice (D): This is the correct answer choice.

Answer choice (E):


This explanation is still in progress. Please post any questions below!
 annrachweila
  • Posts: 8
  • Joined: Sep 12, 2019
|
#72648
Hi,
I managed to narrow down my answer choices to D & E, but couldn’t figure out why D is the better answer. The stimulus says that fingerprints aren’t going to be implemented because they are a lot more expensive and they can only replace passwords if it becomes a standard. So isn’t it assuming that if it’s less expensive than passwords, then it’ll be standardized and replace passwords?
 Paul Marsh
PowerScore Staff
  • PowerScore Staff
  • Posts: 290
  • Joined: Oct 15, 2019
|
#72728
Hi Ann! This is a Justify the Conclusion question. In a typical Justify question, there will be some sort of gap between the premises and the conclusion. The correct answer choice is sufficient to leap over that gap and link the premises to the conclusion.

So first, we want to identify the conclusion. For this stimulus, it's the statement, "Computer passwords are not
going to be replaced by these other options anytime soon."

The premises are 1) Experts say that computer passwords are less secure than other options, 2) Other options are more expensive, and 3) Other options will replace passwords only if they become standard on most computers.

(Side note: if we were to write that last premise as a conditional, what would it look like? It would look like this: If other options replace passwords :arrow: They have become standard. It would NOT be: If other options become standard :arrow: They will replace passwords. Remember that the phrase "only if" always introduces a necessary condition. This can be a tricky concept for many people, so please review Lesson 2 in the LSAT Course Book if this stuff is giving you trouble!)

Now, do those 3 premises lead us 100% logically to our conclusion that passwords will not be replaced by other options anytime soon? No! There are gaps that we can think of. For example, what if the other options become less expensive and standard very soon? There is nothing in our premises that talks about any change being a long way away. Yet that is precisely what the conclusion deals with. Whenever there is a concept or idea in our conclusion that was not present in the premises, that is a clear gap between the premises and the conclusion. In other words, the conclusion does not follow logically from the premises because there is a new concept in the conclusion that was not previously accounted for. A correct Justify answer choice is going to close that gap by explaining the new concept, and therefore lead our premises 100% logically to the conclusion. So here, we are looking for an answer choice that closes the gap by telling us how those alternative security options are not going to be cheap and standard in the near future. (D) does just that.

The other answer choices do not. (A) and (B) do not address the gap. (C) does address the idea of "anytime soon", but the whole answer choice is essentially just a re-statement of the conclusion (it does nothing to fix the gap). (E) also does nothing to tell us why the alternative security options are not getting cheaper and more standard anytime soon, which is what we need in order for the conclusion to follow logically from the premises.

Justify questions can be tricky for a lot of test-takers. Some students struggle with finding the gap between the premises and the conclusion, and others get Justify question stems confused with Assumption question stems. Review lessons 4 and 5 in the LSAT Course books if you find yourself having trouble. Hope that helps!
 gwlsathelp
  • Posts: 93
  • Joined: Jun 21, 2020
|
#91952
Stimulus:
replace passwords :arrow: standard on most computers
(contrapositive: standard on most computers :arrow: replace passwords)
replace passwords
Correct Answer:
security option more expensive :arrow: standard on most computers
How does that fit in with the stimulus? Like so...
security option more expensive :arrow: standard on most computers
contrapositive: standard on most computers :arrow: replace passwords
replace passwords
 Adam Tyson
PowerScore Staff
  • PowerScore Staff
  • Posts: 5153
  • Joined: Apr 14, 2011
|
#92206
Correct! Justify the conclusion by making it the necessary condition in a conditional relationship in which the sufficient condition occurs!
User avatar
 maxtar56
  • Posts: 1
  • Joined: Mar 13, 2024
|
#105654
Paul Marsh wrote: Fri Dec 20, 2019 6:17 pm Hi Ann! This is a Justify the Conclusion question. In a typical Justify question, there will be some sort of gap between the premises and the conclusion. The correct answer choice is sufficient to leap over that gap and link the premises to the conclusion.

So first, we want to identify the conclusion. For this stimulus, it's the statement, "Computer passwords are not
going to be replaced by these other options anytime soon."

The premises are 1) Experts say that computer passwords are less secure than other options, 2) Other options are more expensive, and 3) Other options will replace passwords only if they become standard on most computers.

As for computer security, I would recommend using mfa prices to guarantee the protection of your data from intruders.

(Side note: if we were to write that last premise as a conditional, what would it look like? It would look like this: If other options replace passwords :arrow: They have become standard. It would NOT be: If other options become standard :arrow: They will replace passwords. Remember that the phrase "only if" always introduces a necessary condition. This can be a tricky concept for many people, so please review Lesson 2 in the LSAT Course Book if this stuff is giving you trouble!)

Now, do those 3 premises lead us 100% logically to our conclusion that passwords will not be replaced by other options anytime soon? No! There are gaps that we can think of. For example, what if the other options become less expensive and standard very soon? There is nothing in our premises that talks about any change being a long way away. Yet that is precisely what the conclusion deals with. Whenever there is a concept or idea in our conclusion that was not present in the premises, that is a clear gap between the premises and the conclusion. In other words, the conclusion does not follow logically from the premises because there is a new concept in the conclusion that was not previously accounted for. A correct Justify answer choice is going to close that gap by explaining the new concept, and therefore lead our premises 100% logically to the conclusion. So here, we are looking for an answer choice that closes the gap by telling us how those alternative security options are not going to be cheap and standard in the near future. (D) does just that.

The other answer choices do not. (A) and (B) do not address the gap. (C) does address the idea of "anytime soon", but the whole answer choice is essentially just a re-statement of the conclusion (it does nothing to fix the gap). (E) also does nothing to tell us why the alternative security options are not getting cheaper and more standard anytime soon, which is what we need in order for the conclusion to follow logically from the premises.

Justify questions can be tricky for a lot of test-takers. Some students struggle with finding the gap between the premises and the conclusion, and others get Justify question stems confused with Assumption question stems. Review lessons 4 and 5 in the LSAT Course books if you find yourself having trouble. Hope that helps!
It is clear that in order to justify the conclusion that computer passwords will not be replaced by other options in the near future, we need to bridge the gap between premises and conclusion. The key is to show why these alternative security options will not become cheaper and more standard in the near future.

Answer choice (D) successfully closes this gap by explaining that implementing other options would require large-scale changes to existing computer systems, which are unlikely to happen in the near future due to high cost and complexity. This directly confirms the conclusion drawn from the premises.

I appreciate your clarification on the use of conditional statements and how they apply to these types of questions. To fully understand the logical sequence of an argument, it is important to recognize the necessary conditions introduced by phrases such as “only if.”

Get the most out of your LSAT Prep Plus subscription.

Analyze and track your performance with our Testing and Analytics Package.